1
$\begingroup$

Consider $X_1, X_2, \dots, X_n$ as a random sample from a distribution with the probability density function (pdf):

$$ f(x) = \begin{cases} e^{-(x - \theta)}, & \text{for } x < \theta, -\infty<\theta<\infty \\ 0, & \text{otherwise}. \end{cases}$$

If the estimator for the parameter $\theta$ is $Y_n = \min\{X_1, X_2, \dots, X_n\}$, then is the estimator $Y_n$ unbiased for $\theta$?

I know that it is unbiased if $E(Y_n)=\theta$ and I have found the $f_{Y_n}(x)=-ne^{n(\theta-x)}$. Now what I'm confuse is to find the $E(Y_n)$ what will be the lower and upper bound for the integral $\int_{-\infty}^{\infty}-nxe^{n(\theta-x)}dx$?

$\endgroup$
2
  • 2
    $\begingroup$ Welcome to MSE. It is in your best interest that you type your posts (using MathJax) instead of posting links to pictures. $\endgroup$ Commented Apr 9 at 14:16
  • 2
    $\begingroup$ Is this a valid pdf? It does not seem to integrate to 1 (its integral is infinity?) As $x$ becomes large and negative, the pdf also increases unboundedly. $\endgroup$ Commented Apr 9 at 14:26

1 Answer 1

4
$\begingroup$

It seems that there is a typo in the task; this should be $$ f(x) = \begin{cases} e^{-(x - \theta)}, & \text{for } x \color{red}{>} \theta, -\infty<\theta<\infty \\ 0, & \text{otherwise}. \end{cases} $$ A way is to find the density of $Y_n=\min\{X_1, X_2, \dots, X_n\}$, then compute the expectation.

Alternatively, notice that $X'_i:=X_i-\theta$ has an exponential distribution with parameter $1$. Letting $Y'_n=\min\{X'_1, X'_2, \dots, X'_n\}$, we have $Y_n=\theta+Y'_n$. Thus the biais is $\mathbb E[Y'_n]$, which is not zero.

$\endgroup$
2
  • 2
    $\begingroup$ +1. You could say that $Y_n \ge \theta$ with probability $1$ with a positive probability that $Y_n> \theta$ (in fact also $1$) so you must have $\mathbb E[Y] > \theta$, equivalent to saying $Y'_n \ge 0$ with probability $1$ with a positive probability that $Y'_n> 0$ so you must have $\mathbb E[Y'] > 0$. $\endgroup$ Commented Apr 10 at 10:48
  • $\begingroup$ Thank you. Yes, turns out the question contains a typo, that's why I was confused $\endgroup$ Commented Apr 10 at 14:11

You must log in to answer this question.

Start asking to get answers

Find the answer to your question by asking.

Ask question

Explore related questions

See similar questions with these tags.